ارسال ها
62
لایک ها
24
امتیاز
0
پاسخ : ماراتن نظریه ی اعداد (سطح پیشرفته)

Kتمام توابع
به طوری که برای هر
و هر عدد اول
،
اگر و فقط اگر
.
 
آخرین ویرایش توسط مدیر

m-saghaei

New Member
ارسال ها
338
لایک ها
258
امتیاز
0
پاسخ : ماراتن نظریه ی اعداد (سطح پیشرفته)

Kفرض کنید
سه عدد طبیعی باشند که
نشان دهید دو مجموعه
بر هم منطبق اند.
سوالش فکر کنم تکراریه ها !!
 

AHZolfaghari

Well-Known Member
ارسال ها
935
لایک ها
1,654
امتیاز
93
پاسخ : ماراتن نظریه ی اعداد (سطح پیشرفته)

Kفرض کنید
سه عدد طبیعی باشند که
نشان دهید دو مجموعه
بر هم منطبق اند.
این سوال رو من خودم گذاشتمش . لطفا سوالو عوض کنید
 

حمید آنالیز

Well-Known Member
ارسال ها
1,351
لایک ها
1,322
امتیاز
113
پاسخ : ماراتن نظریه ی اعداد (سطح پیشرفته)

اهم اهم با اجازه من سوالو میزازم
دنباله ی a1,a2,a3,...,an,...از اعداد طبیعی مفروض استبه طوری که anبر 5 بخش پذیر نیستوبرای هر nداریم:an+1=an+bnکهbnآخرین رقم (رقم یکان)anاست.ثابت کنید این دنباله مشتمل بر تعداد نا متناهی توان 2 است.
 

REZA 73

Active Member
ارسال ها
139
لایک ها
184
امتیاز
43
پاسخ : ماراتن نظریه ی اعداد (سطح پیشرفته)

اهم اهم با اجازه من سوالو میزازم
دنباله ی a1,a2,a3,...,an,...از اعداد طبیعی مفروض استبه طوری که anبر 5 بخش پذیر نیستوبرای هر nداریم:an+1=an+bnکهbnآخرین رقم (رقم یکان)anاست.ثابت کنید این دنباله مشتمل بر تعداد نا متناهی توان 2 است.
ابتدا ثابت میکنیم که تو این دنباله یه عدد مضرب 4 هست.
بعد با توجه به این که :

پس برای این که عدد توان دو تولید کنیم کافیه:

در این صورت حتما یک m وجود دارد . :4:
 
آخرین ویرایش توسط مدیر

Dadgarnia

New Member
ارسال ها
1,350
لایک ها
1,127
امتیاز
0
پاسخ : ماراتن نظریه ی اعداد (سطح پیشرفته)

چون سوال قبل به مدت زیادی حل نشده باقی مونده من سوال بعدو میذارم:
همه ی اعداد طبیعی
را بیابید که برای هر
ی صحیح وجود داشته باشد
ای که
.
 

REZA 73

Active Member
ارسال ها
139
لایک ها
184
امتیاز
43
پاسخ : ماراتن نظریه ی اعداد (سطح پیشرفته)

چون سوال قبل به مدت زیادی حل نشده باقی مونده من سوال بعدو میذارم:
همه ی اعداد طبیعی
را بیابید که برای هر
ی صحیح وجود داشته باشد
ای که
.
جواب مساله
میباشد چون به ازای
وجود ندارد به طوری که :
اثبات هم با برهان خلف است :
اما :
پس
که به وضوح غیرممکنه.
به ازای N های دیگه مساله جواب نداره. اگه دوستی راه حل این قسمت رو میخواد بگه بنویسم.

* در ضمن راهنمایی که برای سوال خودم گذاشته بودم چیزی کم از راه حل نداشت. نمیدونم شما چطور میگید سوال به مدت طولانی حل نشده!*
:223:
 
آخرین ویرایش توسط مدیر

mahdi math

New Member
ارسال ها
152
لایک ها
61
امتیاز
0
پاسخ : ماراتن نظریه ی اعداد (سطح پیشرفته)

خب سوال بذارید دیگه
 

Dadgarnia

New Member
ارسال ها
1,350
لایک ها
1,127
امتیاز
0
پاسخ : ماراتن نظریه ی اعداد (سطح پیشرفته)

جواب مساله
میباشد چون به ازای
وجود ندارد به طوری که :
اثبات هم با برهان خلف است :
اما :
پس
که به وضوح غیرممکنه.
به ازای N های دیگه مساله جواب نداره. اگه دوستی راه حل این قسمت رو میخواد بگه بنویسم.

* در ضمن راهنمایی که برای سوال خودم گذاشته بودم چیزی کم از راه حل نداشت. نمیدونم شما چطور میگید سوال به مدت طولانی حل نشده!*
:223:
منظورم این سوال بود:
Kتمام توابع
به طوری که برای هر
و هر عدد اول
،
اگر و فقط اگر
.
که شرت لیست 2007 هست. سوال بعد:
ثابت کنید بی نهایت n صحیح وجود دارد که
خالی از مربع باشد.
 

حمید آنالیز

Well-Known Member
ارسال ها
1,351
لایک ها
1,322
امتیاز
113
پاسخ : ماراتن نظریه ی اعداد (سطح پیشرفته)

منظورم این سوال بود:

که شرت لیست 2007 هست. سوال بعد:
ثابت کنید بی نهایت n صحیح وجود دارد که
خالی از مربع باشد.
ببخشین منظورتونو از سوال متوجه نشدم!میشه بشرحین؟:1:
 

Dadgarnia

New Member
ارسال ها
1,350
لایک ها
1,127
امتیاز
0
پاسخ : ماراتن نظریه ی اعداد (سطح پیشرفته)

ببخشین منظورتونو از سوال متوجه نشدم!میشه بشرحین؟:1:
به یه عدد مثل n میگیم خالی از مربع به طوریکه اگه p یه عامل اول ازش باشه
.
 

TheOverlord

New Member
ارسال ها
159
لایک ها
282
امتیاز
0
پاسخ : ماراتن نظریه ی اعداد (سطح پیشرفته)

لم :

اثبات:

حال فرض کنید
اول باشد، دقت کنید که
حال اگر هر دو همزمان رخ دهند،
اما دقت کنید که
پس
پس
پس
که تناقض است. پس دقیقا یکی از
و
رخ میدهد، پس
. از طرفی بدیهتا
زیرا
.

حال میخواهیم ببینیم برای یک
ثابت، چند عدد مانند
وجود دارد که
و
خالی از مربع نیست. در حقیقت، میبینیم که چند
هستند که
بر یک
- که
اول و ثابت است - بخش پذیر است، سپس تعداد کل اعداد با این خاصیت، طبق اصل شمول و عدم شمول كمتر مساوي جمع این اعداد برای
های مختلف است.
دقت کنید که این عدد برای یک
که ثابت است یا صفر است - هیچ
وجود ندارد که
مانند حالت 2 و 3- یا حداکثر
است زیرا برابر با تعداد اعدادی است که از
کمتر مساویند و به پیمانه
با مثبت یا منفی یک عدد ثابت همنهشتند. حال بدیهتا همه
هايي كه براي آنها
وجود دارد بطوري كه
،كمتر يا مساوي
هستند. بنابرين كافيست
را بيابيم كه حداكثر تعداد اعدادي كه در خاصيت ما صدق نميكنند را بيابيم. دقت كنيد كه اگر
تعداد اعداد اول نابیشتر از
باشد آنگاه
و

حال دقت کنید که همه اعداد به غیر از 2و3و5 به پیمانه 30 همنهشت یکی از اعداد
هستند. پس فرض کنید که
در این صورت
پس
یعنی تعداد اعدادی که مناسب ما نیستند حداکثر
است، پس تعداد اعدادی که مناسب ما هستند حداقل برابر است با
که با زیاد شدن
به هر اندازه زیاد می شود. پس تعداد اعدادی که مناسب ما هستند به هر اندازه زیاد میشود، پس بینهایت تا از این اعداد داریم.

سوال بعد:
همه چند جمله ای های
را بیابید که برای هر
، وجود داشته باشد
که


 
آخرین ویرایش توسط مدیر

حمید آنالیز

Well-Known Member
ارسال ها
1,351
لایک ها
1,322
امتیاز
113
پاسخ : ماراتن نظریه ی اعداد (سطح پیشرفته)

با توجه به گزشتن زمان زیاد از زمان اخرین پست سوالی بعد که من اشکال دارم رو مینویسم
اگر
همگی
ها را پیدا کنید.

---- دو نوشته به هم متصل شده است ----

همه ی pوqوrاولند
 

Dadgarnia

New Member
ارسال ها
1,350
لایک ها
1,127
امتیاز
0
پاسخ : ماراتن نظریه ی اعداد (سطح پیشرفته)

با توجه به گزشتن زمان زیاد از زمان اخرین پست سوالی بعد که من اشکال دارم رو مینویسم
اگر
همگی
ها را پیدا کنید.

---- دو نوشته به هم متصل شده است ----

همه ی pوqوrاولند
فرض مي كنيم
اگه سه تا رابطه رو در هم ضرب كنيم بدست مياد:

حالا داريم:

پس
.
 

حمید آنالیز

Well-Known Member
ارسال ها
1,351
لایک ها
1,322
امتیاز
113

AHZolfaghari

Well-Known Member
ارسال ها
935
لایک ها
1,654
امتیاز
93
پاسخ : ماراتن نظریه ی اعداد (سطح پیشرفته)

یه سوال قشنگ:
همه ی اعداد اول دو به دو متمایز
را بیابید که
.
در اثنای دینی خوندن اونم تا دیروقت ، تصمیم گرفتم رو این سوال فکر کنم !!
حالتی که p,q با هم برابر باشند رو فعلا کنار میذاریم و فرض میکنیم متمایزند و از اینجا به راحتی بدست میاد q از p بزرگ تر هستش .
الان اگه p,q هردو بخوان فرد باشند یعنی طرف راست مطلقا فرد خواهد بود ( درصورت 2 نبودن r ) و طرف چپ زوج پس بلافاصله نتیجه میگیریم که r=2

پس :

پس :

حالا داریم :

پس داریم :

که می بینیم از جایی به بعد غلطه ( ببخشید الان نصفه شبه حوصله ندارم چک کنم اما چون درجه راست 3 و چپ 2 هست معلومه از جایی به بعد بهم میخوره )
پس حالتی که p=q باشه میمونه که در اون صورت p^3 = 2q^2 که یعنی p=q=2 .
پس یه مجموعه جواب : (2,2,r) بقیش طبق رابطه بالا میشه !
خداکنه جوپ نداشته باشه !!
 

حمید آنالیز

Well-Known Member
ارسال ها
1,351
لایک ها
1,322
امتیاز
113
پاسخ : ماراتن نظریه ی اعداد (سطح پیشرفته)

در اثنای دینی خوندن اونم تا دیروقت ، تصمیم گرفتم رو این سوال فکر کنم !!
حالتی که p,q با هم برابر باشند رو فعلا کنار میذاریم و فرض میکنیم متمایزند و از اینجا به راحتی بدست میاد q از p بزرگ تر هستش .
الان اگه p,q هردو بخوان فرد باشند یعنی طرف راست مطلقا فرد خواهد بود ( درصورت 2 نبودن r ) و طرف چپ زوج پس بلافاصله نتیجه میگیریم که r=2

پس :

پس :

حالا داریم :

پس داریم :

که می بینیم از جایی به بعد غلطه ( ببخشید الان نصفه شبه حوصله ندارم چک کنم اما چون درجه راست 3 و چپ 2 هست معلومه از جایی به بعد بهم میخوره )
پس حالتی که p=q باشه میمونه که در اون صورت p^3 = 2q^2 که یعنی p=q=2 .
پس یه مجموعه جواب : (2,2,r) بقیش طبق رابطه بالا میشه !
خداکنه جوپ نداشته باشه !!
خوب رو سوال گفته دو به دو متمایز(انگار دینی خوندن رو دقتتون تاثیر گزاشته!!!)
پس ینی جواب نداریم دیگه.
 

Dadgarnia

New Member
ارسال ها
1,350
لایک ها
1,127
امتیاز
0
پاسخ : ماراتن نظریه ی اعداد (سطح پیشرفته)

در اثنای دینی خوندن اونم تا دیروقت ، تصمیم گرفتم رو این سوال فکر کنم !!
حالتی که p,q با هم برابر باشند رو فعلا کنار میذاریم و فرض میکنیم متمایزند و از اینجا به راحتی بدست میاد q از p بزرگ تر هستش .
الان اگه p,q هردو بخوان فرد باشند یعنی طرف راست مطلقا فرد خواهد بود ( درصورت 2 نبودن r ) و طرف چپ زوج پس بلافاصله نتیجه میگیریم که r=2

پس :

پس :

حالا داریم :

پس داریم :

که می بینیم از جایی به بعد غلطه ( ببخشید الان نصفه شبه حوصله ندارم چک کنم اما چون درجه راست 3 و چپ 2 هست معلومه از جایی به بعد بهم میخوره )
پس حالتی که p=q باشه میمونه که در اون صورت p^3 = 2q^2 که یعنی p=q=2 .
پس یه مجموعه جواب : (2,2,r) بقیش طبق رابطه بالا میشه !
خداکنه جوپ نداشته باشه !!
راه حلتون درسته و اگه اون نامساوي رو هم چك كنيد جواب
بدست مياد ولي اون نامساوي هايي كه نوشتين همشون بايد حالت تساوي هم داشته باشن چون اگه اينجوري نباشه يه جوابي كه سوال داره بدست نمياد.
سوال بعد:
اعدادي صحيح و مثبت هستند به طوريكه
ثابت كنيد
عددي مركب است.
 
بالا